LSAT and Law School Admissions Forum

Get expert LSAT preparation and law school admissions advice from PowerScore Test Preparation.

 Administrator
PowerScore Staff
  • PowerScore Staff
  • Posts: 8917
  • Joined: Feb 02, 2011
|
#35039
Complete Question Explanation

Point at Issue. The correct answer choice is (C)

This two-speaker stimulus presents Ana’s and Pankaj’s views about a proposed smoking ban. Ana opposes the ban because it involves the government preventing people from "doing things that only harm themselves." Meaning she believes the smoking being banned has no external harms (does not impact others).

Pankaj does not explicitly disagree with Ana about whether the ban is justified, but he does encourage her to keep in mind that the ban applies "only to [people] smoking in public places" and would not prohibit them from smoking in private.

In short, Pankaj is suggesting that because the only smoking affected is that which occurs in public settings—i.e. potentially around other people—then Ana's criticism of the ban as a restriction on exclusively self-harming behavior may not be correct. The ban could theoretically be designed to prevent harm to others in public proximity of smokers.

The question stem identifies this as a Point at Issue question, meaning the correct answer choice will be something that both speakers express (or at least clearly imply) an opinion on, and about which their opinions are opposites. Fortunately, as discussed above, we have a great prephrase here: Ana and Pankaj disagree about the rationale behind this smoking ban, and whether its preventative intent is exclusive to just smokers (Ana's position) or not (Pankaj's).

Answer choice (A): As discussed above, Ana would likely say no to this, as she thinks that the government should not restrict people from doing things they alone are harmed by (the fact that A here doesn't include the phrase "harming only themselves" makes it subject to some uncertainty in terms of Ana's views, but as we'll see when we consider Pankaj's position it's irrelevant). However, Pankaj never mentions what the government should or shouldn't do, only what the ban does and who it might be intended to protect. Since we cannot know his position this answer is incorrect.

Answer choice (B): Again, although Ana likely agrees—she describes her view as based on libertarian principles—Pankaj does not address libertarian principles or how the government should behave. So like (A) we can eliminate this choice.

Answer choice (C): This is the correct answer choice. Ana opposes the ban based upon the libertarian rationale that it restricts smokers from behavior that is exclusively harmful to them. In other words, she believes the ban's preventative intent is limited to only the smokers. Without saying whether he disagrees with the ban or not, Pankaj contests that exclusivity: the ban restricts public, not private, smoking, so the ban's intent may be about preventing harm to other people in public near smokers, rather than solely the smokers themselves.

Answer choice (D): We cannot say whether Ana thinks the smoking ban would prohibit smoking in public places, as she never says anything about it. Her view was not dependent on where the smoking ban would be in effect, but only who is affected by it. Since she gives no information on this answer choice it can be eliminated.

Answer choice (E): Neither Ana nor Pankaj addressed the larger issue of whether there are any cases in which the government should attempt to regulate private behavior. Ana’s view related just to the regulation of one’s behavior when the behavior only hurts oneself. And Pankaj only speaks on the nature of this one ban, not on how the government should behave or when certain behavior is/isn't appropriate.
 mankariousc
  • Posts: 32
  • Joined: Feb 13, 2017
|
#35666
Hello:

According to the Answer Key at the end of the test #19 on section 2 is C.
User avatar
 Jonathan Evans
PowerScore Staff
  • PowerScore Staff
  • Posts: 726
  • Joined: Jun 09, 2016
|
#35844
Thank you, mankariousc! You are absolutely right. I have corrected the explanation accordingly.

Get the most out of your LSAT Prep Plus subscription.

Analyze and track your performance with our Testing and Analytics Package.